0
$\begingroup$

I have heard of the following claim “the capacitance matrix of an isolated system of conductors is diagonally dominant” but I couldn’t find a way of proving it myself or any reference for that matter. Does anyone know how to prove it or where to find a rigorous proof?

$\endgroup$

1 Answer 1

0
$\begingroup$

First of all, the article by John Denker is useful as an explanation of capacity matrices.

Suppose we have two capacitors, one with a capacitance of 10 µF and the other with a capacitance of 20 pF. Arrange them as shown in the figure and join the bottom conductors of the two capacitors to make a three-conductor problem. (The original picture is taken from Wikipedia(jpn)).

3 conductors problem

Symbolize capacitance as $C_A(=10\mu F), C_B(=20pF)$, and assign conductor numbers as shown in the figure. The capacitance matrix for a three-conductor system is as follows, \begin{equation*} C= \begin{pmatrix} C_{11} & C_{12} & C_{13} \\ C_{21} & C_{22} & C_{23} \\ C_{31} & C_{32} & C_{33} \end{pmatrix} = \begin{pmatrix} C_A & -C_A & 0 \\ -C_A & C_A+C_B & -C_B \\ 0 & -C_B & C_B \end{pmatrix} \end{equation*} Since conductors 1 and 3 are not coupled, the corresponding component ($C_{13}$) is set to 0. We chose $C_{33}$ as the representative of the diagonal component and $C_{12}$ as the representative of the off-diagonal component, and calculated the ratio of their magnitudes. \begin{equation*} \text{ratio}=\frac{-C_{12}}{C_{33}}=\frac{10\mu F}{20pF}=5.0\times 10^5 \end{equation*}

Therefore, the statement "the capacitance matrix of an isolated system of conductors is diagonally dominant" is false.

$\endgroup$
2
  • $\begingroup$ I am sorry but I couldn’t follow why the matrix is not diagonally dominant. It seems to me that the definition is satisfied (en.m.wikipedia.org/wiki/Diagonally_dominant_matrix where I am considering the weak definition). $\endgroup$ Commented Jul 28, 2023 at 15:19
  • $\begingroup$ @Matteo Menghini, I did not know that the term diagonal dominance of a matrix had a rigorous definition. I thought that if the diagonal component was small enough compared to the non-diagonal terms, then it was not diagonally dominant. My description must be wrong. My apologies. $\endgroup$
    – HEMMI
    Commented Jul 28, 2023 at 22:14

Your Answer

By clicking “Post Your Answer”, you agree to our terms of service and acknowledge you have read our privacy policy.

Not the answer you're looking for? Browse other questions tagged or ask your own question.